0A library bought b books for its collection.

This topic has expert replies
Moderator
Posts: 7187
Joined: Thu Sep 07, 2017 4:43 pm
Followed by:23 members
0A library bought b books for its collection. What was the total amount of money required to buy the b books?

(1) b = 40

(2) Each of the books cost the library at least $12.50

Which statement is the best option?

OA E

User avatar
Legendary Member
Posts: 2663
Joined: Wed Jan 14, 2015 8:25 am
Location: Boston, MA
Thanked: 1153 times
Followed by:128 members
GMAT Score:770

by DavidG@VeritasPrep » Sun Nov 26, 2017 7:30 am
lheiannie07 wrote:0A library bought b books for its collection. What was the total amount of money required to buy the b books?

(1) b = 40

(2) Each of the books cost the library at least $12.50

Which statement is the best option?

OA E
The amount of money required to buy the b books = b * cost per book.

Statement 1: we've got 40 books, but we have no idea how much each book costs. Not sufficient.

Statement 2: Nothing about the number of books. Not sufficient.

Together: We know there are 40 books, but the cost per book could be exactly $12.50, or it could be $14 or $15, etc. Because we don't know the cost per book, the statements together are not sufficient to answer the question. The answer is E
Veritas Prep | GMAT Instructor

Veritas Prep Reviews
Save $100 off any live Veritas Prep GMAT Course